0

Hey...
Ich soll zeigen, dass die Funktion
\(f: [0,\infty) \rightarrow [0,\infty) \\ x \rightarrow x^n \\ \)

für jedes \(nε\mathbb{N}\) eine Umkehrfunktion besitzt, die stetig und streng monoton wachsend ist.
Die Umkehrfunktion ist bereits bekannt: \(\sqrt[n]{x}\)

Wie kann ich zeigen, dass die Umkehrfunktion für jedes n streng monoton wachsend ist?
Dankeschön..

gefragt
inaktiver Nutzer

 
Kommentar schreiben
1 Antwort
0

Ich zittiere hier von aleph1.info:

"Ist f(x) < f(x′) für x, x′  ∈  I, so ist x < x′, da f monoton steigt. Aber es gilt x = f−1(f(x)) und x′ = f−1(f(x′)). Also ist f−1 streng monoton steigend.

Sei q = f(p)  ∈  Q beliebig. Wir zeigen, dass f−1 : Q   ε-δ-stetig in q ist. Sei hierzu ε > 0. Wir suchen ein δ > 0 mit
∀y  ∈  Q (|y − q| < δ    |f−1(y) − p| < ε).

1. Fall:  p ist kein Randpunkt des Intervalls I

Durch etwaige Verkleinerung von ε können wir annehmen, dass [ p − ε, p + ε ] ⊆ I. Sei nun δ > 0 so klein, dass
] q − δ, q + δ [  ⊆  ] f(p − ε), f(p + ε) [.

Dann ist δ wie gewünscht. Denn sei y  ∈  Q mit |y − q| < δ. Dann gilt
f(p − ε)  <  y  <  f(p + ε).

Da f−1 streng monoton ist, gilt also p − ε < f−1(y) < p + ε.

2. Fall:  p ist der linke Randpunkt des Intervalls I, d. h., q = min(Q)

Wir können I ≠ { p } und [ p, p + ε ] ⊆ I annehmen. Sei δ > 0 so klein, dass
[ q, q + δ [  ⊆  [ q, f(p + ε) [.

Ist nun y  ∈  Q mit |y − q| < δ, so ist y ≥ q und f(p) ≤ y < f(p + ε).

Da f−1 streng monoton ist, gilt also p ≤ f−1(y) < p + ε.

3. Fall:  p ist der rechte Randpunkt des Intervalls I, d. h., q = max(Q)

Analog zum zweiten Fall."

Diese Antwort melden
geantwortet

Punkte: 85

 

Kommentar schreiben